LSAT and Law School Admissions Forum

Get expert LSAT preparation and law school admissions advice from PowerScore Test Preparation.

 nicholaspavic
PowerScore Staff
  • PowerScore Staff
  • Posts: 271
  • Joined: Jun 12, 2017
|
#42323
Hi biskam,

I agree with the explanation above but also draw your attention to the extreme language used in Answer Option (D). "Never?" That is a big red flag and there is no indication that the stimulus thinks that.

Thanks for the great question and let us know if this clarifies it for you.
 lilmissunshine
  • Posts: 94
  • Joined: Jun 07, 2018
|
#47116
Hello,

Could someone explain why (D) is incorrect again? Sorry I don't think I full understand the explanation below.
Ricky_Hutchens wrote: D actually says that the argument ignores the possibility that how long a patient stays in the hospital might be relevant to his recovery. However, the argument doesn't do this because it is comparing recovery rates for similar patients with similar illnesses.
Thanks a lot!
User avatar
 Dave Killoran
PowerScore Staff
  • PowerScore Staff
  • Posts: 5852
  • Joined: Mar 25, 2011
|
#47142
Sure, happy to help!

The initial concern over (D) revolves around the strength of the language used. Does the author really presume that the length of time is never relevant to recovery rates? No, I'd say that is too strong and for me knocks (D) out of contention immediately.

Why is it the case that never is too strong? Because the author clearly is using hospital stay lengths to make a judgment about University Hospital. Because the lengths are different, and recovery rates are similar at both hospitals for the same illnesses, the author concludes that University Hospital could shorter stay times with no harm. But this is a questionable judgment because we don't know how many patients had similar illnesses. What if the two hospitals generally handle entirely different conditions, and that "overlap" covers only 5 patients? That's what (C) suggests, and why (C) is correct.

Please let me know if that helps. Thanks!
 lilmissunshine
  • Posts: 94
  • Joined: Jun 07, 2018
|
#47154
Thanks a lot Dave!
 jaldred
  • Posts: 2
  • Joined: Jan 09, 2019
|
#61903
So I am having trouble seeing that C is correct. The stimulus states "studies show that recovery rates at the two hospitals are similar for patients with similar illnesses". To me this infers that there are AT LEAST SOME patients at both hospitals with similar illnesses. If this is not true, how could the studies have been conducted? With this established, a reduction in length of stay in even just a single patient would mathematically reduce the average length.
 James Finch
PowerScore Staff
  • PowerScore Staff
  • Posts: 943
  • Joined: Sep 06, 2017
|
#61916
Hi J. Alred,

It sounds like you're assuming that patients with similar illnesses stay longer at University Hospital than Edgewater. We don't actually know this, only that the average stay overall is longer at Edgewater. Patients that are treated for a broken leg, for example, may stay the exact same length of time at the two hospitals, or even get discharged faster at Edgewater.

While you are correct that there must be some overlap, the problem is that we don't know the proportion of the patients staying at the two hospitals being treated for the same illnesses, and thus the proportion who should be able to be sent home in roughly the same timeframe. Without that information, we can't properly draw the conclusion. Edgewater is potentially a trauma hospital that deals with more severe illnesses and injuries than University Hospital, which leads to longer average stays. (C) represents this logical gap, making it the correct answer choice.

Hope this clears things up!
User avatar
 elizabeth28
  • Posts: 2
  • Joined: Feb 10, 2021
|
#84036
Hello! Can someone help explain why A is wrong?

Thank you!
 Robert Carroll
PowerScore Staff
  • PowerScore Staff
  • Posts: 1787
  • Joined: Dec 06, 2013
|
#84046
Elizabeth,

The stimulus seems to think that University Hospital could decrease the average length of stay without affecting quality of care, so the author seems committed to an idea that at least partially breaks apart length of stay and quality of care. Far from equating them, the author seems to think that one can change without changing the other, so, whatever else the author did, answer choice (A) is not a mistake the author made.

Robert Carroll
User avatar
 elizabeth28
  • Posts: 2
  • Joined: Feb 10, 2021
|
#84071
Right, I understand what it means now. Thank you, Robert!

Get the most out of your LSAT Prep Plus subscription.

Analyze and track your performance with our Testing and Analytics Package.